Download as docx, pdf, or txt
Download as docx, pdf, or txt
You are on page 1of 65

Đề 19

I. Find the word whose bold part is pronounced differently from that of the others in each group. Write your answer (A,
B, C or D) in the space provided on the right. (2.5 pts)
1. A. hasty B. nasty C. tasty D. wastage _________________
2. A. money B. notice C. glance D ankle _________________
3. A. change B. anger C. bridge D. manage _________________
4. A. woman B. home C. phone D. most _________________
5. A. bother B. sunbathe C. nothing D. within _________________
II. Find the word with a different stress pattern in each group.
1. A. encourage B. financial C. departure D. average ________________
2. A. creative B. sensitive C. ambitious D. attentive ________________
3. A. preservative B. congratulate C. preferential D. development ________________
4. A. open B. happen C. offer D. begin ________________
5. A. eternal B. literate C. pessimist D. vulnerable ________________
Write your answer (A, B, C or D) in the space provided on the right. (2.5 pts)
PART TWO : VOCABULARY AND GRAMMAR
I. Choose the word or phrase that fits each sentence.
Write your answer (A, B, C or D) in the space provided on the right. (5 pts)
1. Despite a lot of hardship, the Green City Project will go...........
A. before B. forward C. advance D. ahead ____________
2. After hours of bargaining with the salesman, Jake bought the jacket
for a .........of the original price.
A. fraction B. piece C.part D. spot ____________

3. When the first Chinese restaurants opened in Greece, it was


very difficult to get fresh........of Chinese vegetables
A. provisions B. materials C. supplies D. ingredients ____________
4. I find mending old socks incredibly........ that's why I always ask my
mother to do it for me.
A. hilarious B. tedious C. furious D. recreational ____________
5. Many countries have now succeeded in _______ the malarial mosquito.
A. eradicating B. abolishing C. obliterating D. demolishing __________
6. …………..one day by a passing car, the dog never walked proper again.
A .Having injured C. Injured
B. Injuring D. To be injured ____________
7. He had changed so much since the last time we met that I ……him.
A. could recognize C. wouldn't have recognized
B. could hardy recognize D. don't recognize ____________
8. It was felt that he lacked the …………… to pursue a difficult task
to very end.
A. persuasion B. commitment C.engagement D.obligation ____________
9. Had it not been for the intolerable heat in the hall, they …………
…….much longer.
A. will stay C. would be staying
B. would stay D. would have stayed ____________
10. In front of us ………………………with a beard and blue eyes.
A. is a big man standing C. is standing a big man
B. stand a big man D. a big man is standing ____________
Reading 1. Read the following passage and choose the correct answer for each gap.
Write your answer (A, B, C or D) in the space provided on the right. (5 pts)
The latest addiction to trap thousands of people is Internet, which has been (1) ...... for broken relationships, job losses,
financial ruin and even one suicide. Psychologists now recognize Internet Addiction Syndrome (IAS) as a new illness that
could (2) ............... serious problems and ruin many lives. Special help groups have been set up to (3) ........... sufferers help and
support.
IAS is similar to (4) ...... problems like gambling, smoking and drinking : addicts have dreams about Internet; they need to
use it first thing in the morning; they (5) ............... to their partners about how much time they spend online; they (6) .........
they could cut down, but are unable to do so . A recent study found that many users spend up to 40 hours a week on the
Internet; (7) .............. they felt guilty, they became depressed if they were (8) .............. to stop using it.
Almost anyone can be at risk. Some of the addicts are teenagers who are already hooked on computer games and who
(9).............. it very difficult to resist the games on the Internet. Surprisingly, however, psychologists (10) .......... that most
victims are middle-aged housewives who have never used a computer before.
1. A. accused B. mistaken C. blamed D. faulted _______
2. A. take B. cause C. affect D. lead ______
3. A. recommend B. offer C. suggest D. advise ______
4. A. others B. another C. the other D. other _______
5. A. lie B. cheat C. deceive D. betray _______
6. A. rather B. want C. prefer D. wish ______
7. A. unless B. without C. although D. despite _______
8. A. made B. allowed C. let D. had ______
9. A. have B. find C. feel D. say ______
10. A. say B. tell C. object D. promise _______
Reading 2. Read the passage and answer the questions that follow.
Write your answers in the spaces that follow (5 pts)
The response of most animals when suddenly faced with a predator is to flee. Natural selection has acted in a variety of ways
in different species to enhance the efficacy of the behaviours, known as "flight behaviours" or escape behaviours that are used
by prey in fleeing predators. Perhaps the most direct adaptation is enhanced light speed and agility.
Adaptations for speed, however, are likely to require sacrifices biter attributes, so we might expect only some species to adopt
a simple fast flight strategy.
Another way of enhancing the effectiveness of flight is to move in an erratic and unpredictable way. Many species, like
ptarmigans, snipes, and various antelopes and gazelles, flee from predators in a characteristic zigzag fashion. Rapid
unexpected changes in flight direction make it difficult for a predator to track prey.
In some species, like the European hare, erratic zigzag flight might be more effective in the presence of predators that are faster
than they are and straight light more effective against predators that are slower. One observation that supports this suggestion
is the recorded tendency for slow-flying black-beaded gulls, which are normally able to escape predators by means of direct
flight, to show frequent changes in flight direction when they spot a peregrine falcon (peregrines are adept at capturing flying
birds).
A quite different way of enhancing escape by flight is to use so-called "flash" behaviour. Here, the alarmed prey flees for a
short distance and then "freezes." Some predators are unexcited by immobile prey, and a startling flash of activity followed by
immobility may confuse them. "Flash" behaviour is used in particular by frogs and orthopteran insects, which make
conspicuous jumps and then sit immobile. In some species, "flash" behavior is enhanced by the display of bright body
markings. Good examples of insects with colorful markings are the red and yellow under wing moths. At rest, both species are
a cryptic brown color. When they fly, however, brightly colored hind wings are exposed, which render the moths highly
conspicuous. Similarly, some frogs and lizards have brightly colored patches or frills that may serve a 'flash" function when
they move quickly. Some species even appear to possess "flash" sounds. The loud buzzing and clicking noises made by some
grasshoppers when they jump may serve to emphasize the movement.
1. The description of the prey's movement as "zigzag" suggests that the movement is...........
A. reliable B. fast C. constant D. unpredictable
2. It can be inferred from the passage that the European hare.........
A. is faster than most of its predators
B. is capable of two kinds of flight
C. is more likely to escape using straight flight
D. is preyed upon by gulls and falcons
3. The behavior of black-beaded gulls is most comparable to that of.......
A. gazelles B. European hares C. peregrine falcons D. frogs
4. It can be inferred that black-beaded gulls change direction when they spot a peregrine falcon for which of the following
reasons?
A. The falcons are faster than the gulls.
B. The gulls want to capture the falcons.
C. The falcons are unpredictable.
D. The gulls depend on the falcons for protection.
5. Why does the author mention "grasshopper" in the last line?
A. To contrast animals that "flash" with animals that "freeze".
B. As an example of an animal whose "flash" behaviour is a sound.
C. To compare the jumping behaviour of insects and reptiles.
D. As an example of a predator that moths escape by using "flash" behavior.
Reading 3. Read the passage and answer the questions that follow.
Write your answers in the spaces that follow (10 pts)
ADVERTISING SHIFTS FOCUS
The average citizen is bombarded with TV commercials, posters and newspaper advertisements wherever he goes. Not only
this, but promotional material is constantly on view, with every available public space from shop to petrol station covered with
advertising of some kind. People who are foolish enough to drive with their windows open are likely to have leaflets
advertising everything and anything thrust in at them. The amount of advertising to which we are exposed is phenomenal, yet
advertisers are being hurt by their industry's worst recession in a decade and a conviction that is in many respects more
frightening than the booms and busts of capitalism: the belief that advertising can go no further. Despite the ingenuity of the
advertisers, who, in their need to make their advertisements as visually attractive as possible, often totally obscure the message,
the consumer has become increasingly cynical and simply blanks out all but the subtlest messages. The advertising industry
has therefore turned to a more vulnerable target: the young.

The messages specifically aimed at children are for toys and games - whose promotional budgets increased fivefold in the
1990s - and fast food, which dominates the children's advertising market. Advertisers acknowledge that the commercial
pressures of the 1990s had an extraordinary effect on childhood: it is now generally believed that the cut-off point for buying
toys has been falling by one year every five years. Research, suggests that while not so many years ago children were happy
with Lego or similar construction games at ten or eleven, most of today's children abandon them at six or seven. In effect, the
result is the premature ageing of children.

There is nowhere where the advertising industry's latest preoccupation with the young is so evident as in schools.
Increasingly low budgets have left schools vulnerable to corporate funding and sponsorship schemes in order to provide much
needed equipment, such as computers, or to enable them to run literacy schemes. While on the face of it this would seem to be
a purely philanthropic gesture on the part of the companies concerned, the other side of the coin is a pervasive commercial
presence in the classroom, where textbooks and resource books are increasingly likely to bear a company logo.
This marked shift in advertising perceptions also means that a great deal of supposedly adult advertising has an infantile
appeal, inasmuch as adult products can be presented within an anecdote or narrative, thus making the message more accessible
to young teenagers and smaller children. Children obviously cannot buy these things for themselves; what is behind these
advertisements is more subtle. Advertisers have come to recognize that if children can successfully pester their parents to buy
them the latest line in trainers, then they can also influence their parent's choice of car or credit card, and so children become
an advertising tool in themselves.
There are many, on all sides of the ideological spectrum, who would argue that advertising has little influence on children, who
are exposed to such a huge variety of visual images that advertisements simply become lost in the crowd. Rather, they would
argue that it is the indulgent parents, who do not wish their children to lack for anything, who boost sales figures. While there
may be a great deal of truth in this, it would seem that to deny that advertising influences at all because there is so much of it,
while accepting that other aspects of life do have an effect, is a little disingenuous. In fact, the advertising industry itself admits
that since peer pressure plays such an important role in children's lives, they are not difficult to persuade. And of course, their
minds are not yet subject to the advertising overload their parents suffer from. The question that arises is whether indeed, we as
a society can accept that children, far from being in some sense protected from the myriad of pressures, decisions and choices
which impinge on an adult's life, should now be exposed to this influence in all aspects of their lives, in ways that we as adults
have no control over. Or do we take the attitude that, as with everything else from crossing city streets to the intense
competition of the modern world, children will have to learn to cope, so the sooner they are exposed the better?
1. What does the writer say about advertising in the first paragraph?
A. Capitalism has led to the demise of advertising. B. We should have a cynical view of advertisers.
C. Advertising is facing new challenges these days. D. The industry has run out of new ideas.
2. The bombardment of advertisements has led to ________.
A. children taking more notice of them B. greater difficulty in attracting consumers' attention
C. more appealing advertisements D. people being less likely to spend money
3. How have children changed during the past decade?
A. They have become consumers. B. They are growing up more quickly.
C. They are becoming cleverer. D. They are not playing as much.
4. Which of the following square brackets [A], [B], [C], or [D] best indicates where in the paragraph the sentence “However,
the main thrust of advertising in this area is no longer towards traditional children's products.” can be inserted?
[A] The messages specifically aimed at children are for toys and games - whose promotional budgets increased fivefold
in the 1990s - and fast food, which dominates the children's advertising market. [B] Advertisers acknowledge that the
commercial pressures of the 1990s had an extraordinary effect on childhood: it is now generally believed that the cut-off
point for buying toys has been falling by one year every five years. [C] Research, suggests that while not so many years
ago children were happy with Lego or similar construction games at ten or eleven, most of today's children abandon them
at six or seven. In effect, the result is the premature ageing of children. [D]
A. [A] B. [B] C. [C] D. [D]
5. Which of the following sentences best expresses the meaning of the underlined sentence in paragraph 3?
A. The advertising industry's latest obsession with young people is rather obvious in schools.
B. Nowhere else can we see the advertising industry's latest products for the young as in schools.
C. Schools are places where the advertising industry's latest concern with youngsters is the least obvious.
D. It is in schools that the advertising industry's latest concern with youngsters is the most clearly seen.
6. What does the writer imply in the third paragraph?
A. Advertising agencies need to preserve their reputations. B. Schools welcome aid from big business.
C. There are restrictions on how financial aid may be used. D. Companies expect nothing in return for their help.
7. How have children changed the face of advertising?
A. Children are influencing the purchases of adult products. B. They are now the advertising industry's sole market.
C. More products have to be sold to children. D. Children have become more selective in their choices.
8. The word “who” in the last paragraph refers to ________.
A. many people B. the crowd C. parents D. children
9. What does the writer suggest in the last paragraph?
A. Adults feel increasingly threatened by advertising.
B. Children are unlikely to be influenced by their friends.
C. Parents avoid spending too much money on their children.
D. Children have a less sheltered existence than they used to.
10. In the text as a whole, the writer's purpose is to ________.
A. explain the inspiration for advertisements B. expose the exploitation of children
C. deter parents from giving in to advertisers D. prevent advertisers from infiltrating schools
III. In about 150-200 words, write a paragraph about the importance of examinations (10 pts)
PART FOUR: WRITING
I. Finish each of the sentences below in such a way that it means the same as the original sentence. (10 pts)
1. I assumed that she would learn how to take shorthand after this course.
 I took it ______________________________________________________
2. We regret to inform you that your application has not been successful.
 Much to ______________________________________________________
3. I left without saying goodbye as I didn’t want to disturb the meeting.
 Rather _______________________________________________________
4. Only if you work hard now have you any chance of success.
→ Your chance___________________________________________________
5. You should admit that you are to blame, not to conceal it.
I’d rather__________________________________________________
6. Sharah said to her teacher she was sorry she didn’t finish her exercise.
Sharah apologized ……………………………………………………………………
7. No matter how hard he tried he couldn’t get used to driving the car on the left.
Try……………………………………………………………………………………
8. It is thought that the Prime Minister is considering raising taxes on alcohol and cigarettes.
The Prime Minister is ……………………………………………………..
9. She encouraged me and I applied for this job.
Had ………………………………………………………………………………..
10. If you get to know Sarah more, you‘ll find her more interesting.
The more ……………………………………………………………………….
II. Rewrite each of the sentences below in such a way that its meaning stays the same, using the word given in capitals. Do
not change this word in any way. (5 pts)
1. Jenny didn’t feel like going to the party. MOOD
_________________________________________
2. Did Pamela say why she was so late? REASON
_________________________________________
3. The firm is going to raise everybody’s salary. Given
_________________________________________
4. This contract is not binding until we both sign it BOUND
_________________________________________
5. As far as I know he is still working in Bristol. Knowledge _________________________________________
PART THREE –READING
I. Fill in each of the numbered blanks in the passage below with ONE suitable word.
Write your answers in the spaces that follow. (5 pts)
SAFE CAMPING
Camping in the country is usually great fun, but sometimes things can go wrong. Accidents can happen, so it is
essential to think about safety (1) _________ before you go and while you are there. This will prevent your fun camping trip
turning into (2) _________ less pleasant.
Firstly, you need to plan ahead. Check out the weather forecast a few days in (3) ________ and watch out for any
reports of fires in the area you are thinking of going to. (4) ________ an emergency kit in case you or anyone with you has an
accident or illness while you are there. Choose your camp site carefully, avoiding any places (5) ________there is risk of
flooding before you put up your tent, make (6) ________there are no sharp objects on the ground, or ants’ or wasps’ nests
nearby.
In order to keep insects out of the tent, close it whenever you go in or out. If you need a camp fire for cooking, be
(7)________ not to build it anywhere near your tent, and before you go to bed, remember to put it out completely, preferably
(8)_______ lots of water.
After meals, pick up any bits of food that (9) ________ be left on the ground, as these can attract insects – or larger
creatures. It also makes senses, for the same reason, to keep unused food in closed containers away from the camp. You don’t
want a hungry bear or another (10) _______ suddenly appearing in your tent!
III. Read the following text which contains 10 mistakes. Identify the errors and write the corrections in the corresponding
numbered boxes. (20 pts)
Example: Line 1: ever => never
Line
1 The market for tourism in remote areas is booming as ever before. Countries all across the
2 world are active promoting their 'wilderness' regions – such as mountains, Arctic lands, deserts,
3 small islands and wetlands – to highly spending tourists. The attraction of these areas is obvious: by
4 defining, wilderness tourism requires little or no initial investment. But that does not mean that there
5 is no cost. Like the 1992 UN Conference on Environment and Development recognized, these
6 regions are fragile (i.e. highly vulnerable of abnormal pressures) not just in terms of the culture of
7 their inhabitation. The three most significant types of fragile environment in these respects are
8 deserts, mountains and Arctic areas. An important character is their marked seasonality.
9 Consequently, most human acts, including tourism, are limited to clearly defined parts of the year.
10 Tourists are drawn to these regions by their natural beauty and the unique culture of its
11 people. And poor governments in these areas have welcomed the 'adventure tourists', grateful for
12 the currency they bring. For several years, tourism is the prime source of foreign exchange in Nepal
13 and Bhutan. Tourism is also a key element in the economics of Arctic zones such as Lapland and
14 Alaska and in desert areas such as Ayres Rocks in Australia and Arizona’s Monument Valley.
15
16
II. Complete each of the following sentences using the correct form of a phrasal verb from the box.( 10pts )
live on look through pass down set off turn down
close down come back deal with get up keep up with
1. They offered her a trip to Europe but she it .
2. He has the report and found nothing interesting.
3. Phong walks too fast and it’s really hard to him.
4. These traditional stories have been from parents to children over many generations.
5. My sister is an early bird. She at 7 o’clock every day of the week.
6. We’ll for Tay Ho village at 9 o’clock and arrive at 10 o’clock.
7. When I was a student, I small allowances from my parents.
8. The government must now the preservation of traditional craft villages.
9. The banks have a lot of branches in villages over the last few years.
10. Many foreign tourists decided to Viet Nam for another holida
PART TWO : GRAMMAR AND VOCABULARY
Supply the correct form of the words given in brackets to complete each of the sentences below. Write your answer in
the space provided on the right. (5 pts)

1. Jenny has sent me a very _____ (APOLOGY) letter explaining why she didn’t do what she
promised. ______________
__________
2. People used to suffer from their life-time physical ________(NORMAL)
______________
__________
3. If you think I am going to marry you, then I’m afraid you’re very much ____ (TAKE).
______________
_________
4. The man was found guilty of fraud and sentenced to three years _____(PRISON )
______________
_________
5. He won the discus event at the Olympic Games but was later _____( QUALIFY ) when a medical
check proved that he had been taking drugs
______________
__________
6. The recent hurricane caused ___ ( WIDE) damaged ______________
__________
7. Jim is one of the most ___ (SPEAK ) members of the committee ______________
_________
8. We try to ensure the ____( BE) of our employees ______________
__________
9. The ________ (PAGE) of this document is wrong! There’s no page 13 ______________
__________
10. It’s essential that the application forms be sent back before the _......(LINE)
______________
__________

Đề 20
Fill each gap in the sentences with the correct tense or form of one of the phrasal verbs.
Write your answer (A, B, C or D) in the space provided on the right. (10 pts)
throw away throw out try out
turn back turn down turn off turn on
turn out turn over turn up
1.Could we_____the bike before we buy it? _________
2.He ate the last biscuit and______the packet. _________
3.He was_____of the pub for starting a fight. ________
4. Don’t forget to _____the TV ____before you go out. ________
5.Guess who ______at the party! ________
6.After driving for an hour, we had to _because we realised we
had forgotten to take our passport. ________
7.The teacher told us to _______the page and begin the next exercise. ________
8.Thank you for your offer, but I’m fraid I’ll have to______it_____. ________
9.It’s dark, why haven’t you ______the lights yet? _______
10.If I had known the cake would have ______like this, I never would attempted to make it.
II. Supply the correct form of the words given in brackets to complete the sentences.
Write your answer in the numbered blanks that follow. (5pts)
1.Have you read the latest --------about Madonna’s private life? __________________ ( REVEAL)
2.Mary decided to go on a ________ course. ______________________ (SECRETARY)
3.This matter is very -----------Don't discuss it outside the office. __________________(CONFIDENCE)
4.You have some good ideas but your work is very ________. __________________(ORGANISE)
5.His drug --------- results in his ten years in prison. __________________(TRAFFIC)
6.Don’t try to do things too quickly. Set yourself ______ targets. (REAL)

7.She is a highly ___________woman. (EDUCATION)

8.Be _________ We can’t afford to have children just yet. (PRACTICE)

9. She’s got a very________ mind. (PERCEIVE)

10.We always have a bed ready in the spare room in case visitors arrive _____. (EXPECT)
The passage below contains 10 errors. IDENTIFY and CORRECT them. Write your answers in the spaces that follow.
(0) has been done as an example. (10pts)
AS OLD AS YOU FEEL
1 It might after all be true that you are only as old as you feel. A British clinic is carrying out new high-tech tests to
calculate the “real” biology age of patients based on the rate of physical deterioration. Information on every aspect of a
patient’s health, fitness, lifestyle and family medical history are fed in to a computer to work out whether they are
older or younger to their calendar age suggests.
5 The availability and increasing accuracy of the tests has prompted one leading British gerontologist to call for
biological age to be used to determine when workers should retire. He argues that if an employee’s biological or “real”
age is shown, for example, be 55 when he reaches his 65 th birthday, he should encourage to work for another decade.
Apparently most employers only take into account a person’s calendar years, and two may differ considerably. Some
of those prepared to pay a substantial sum of money for the examinations will be able to smugly walk away with
10 medical evidence shows that they really are as young as they feel, giving them the confidence to act and dress as if
they were younger. Dr Lynette Yong, resident doctor at the clinic that the tests are offered claims that the purpose of
these tests will be to motivate people to improve their health.
The concept of “real” age is set to become big business in the USA with books and websites helping people work out
when their body is older or younger than their years. Other firmly believe that looks will always be the best indicator
15 of age.

Fill each gap in the following passage with a suitable word.


Write your answer in the numbered blanks that follow. (5 pts)

STRESS
Stress is often called a 21st century illness but it has always been with us if perhaps with different names. These days
we regard stress (1) ________ a necessary evil of modern living. Yet stress is not negative and without (2) ________ we
would not enjoy some of the highpoints in life (3) ________ as the anticipation before a date or the tension leading up to an
important match. All these situations produce stress but (4) ________ you can control it and not the other way around, you
will feel stimulated, not worn out. However, (5) ________ these situations, which are generally positive and easier to deal (6)
________, sitting in a train that is running late, (7) ________ stuck in a traffic jam or working to a tight deadline are much
harder to manage and control and can be a significant (8) ________ of stress.
Stress is now recognised as a medical problem and as a significant factor (9) ________ causing coronary heart
disease, high blood pressure and a high cholesterol count. Patients are often unwilling to admit to stress problems since they
feel they are a form of social failure and it is important that symptoms are identified in order to (10) ________ unnecessary
suffering.
In about 150 -200 words, write an essay about the causes of the fact that families now are not as closely-knit as they were
in the past.
The following prompts might be helpful to you.
-Busy lifestyles.
-Modern houses and technology.
-Different viewpoints.
Rewrite each of the sentences using the word given. Do not change this word in any way. (5 pts)
1.Perhaps Brian went home early, (MAY)
2, To Alan's amazement, the passport office was closed when he arrived ( FIND)
1. I am totally convinced that our team will win. ( CHANCE)
2. It's a good thing you wrote the letter or we wouldn't have known what happened. (YOU)
3. My watch was so badly damaged that it wasn’t worth keeping. (POINT
Rewrite each of the following sentences in such a way that it means exactly the same as the sentence   printed before it.
(10 pts)
1.Someone has suggested the resignation of the minister.
It _______________________________________________________________________________________
2.I would prefer him not to have said all those embarrassing things about me.
I’d rather__________________________________________________________________________________
3.Since she couldn’t get through to the police, she ran next door for help.
Not ______________________________________________________________________________________
4.I was about to leave when my aunt came.
I was on the_______________________________________________________________________________
5."You should have waited for us," the team leader said to John.
The team leader criticized____________________________________________________________________
6. I am always short of time.
I never ________________________________________________________
7. His jealousy increases with his love for her.
The more________________________________________________________
8. The result of the entrance exam was never in doubt.
At no time ______________________________________
9. Most students ignored what the teacher was saying.
Few_________________________________________
10. They tried hard so that they would pass the exam.
With____________________________________________
Reading I.
Read the following passage and mark the letter A, B, C, or D on your answer sheet to indicate the correct answer to each of
the questions
Vietnamese generally shake hands when greeting and parting. Using both hands shows respect as does a (1) _____ bow of the
head. In rural areas, elderly people do not extend their hand, they are greeted with a bow. Women are more (2) _____ to bow the
head than to shake hands. Vietnamese names begin with the family name and are (3) _____ by a given name. For example, in the
name Nguyen Van Due, Nguyen is the family name. People address (4) _____ by their given names , but add a title that indicates
their perceived (5) _______ to the other person. These titles are family related rather than professional. Among colleagues, for
example, the (6) _____ of the two might combine the given name with the title of Anh ("Older Brother"). A(n) (7)_____ greeting
combined with the given name and title is Xin chao (“Hello”). Classifiers for gender and familiarity are also combined with the
greeting. In formal meetings, business cards are sometimes (8) _____ on greeting.
Vietnamese people have a strong (9) _____ of hospitality and feel embarrassed if they cannot show their guests full respect by
preparing for their arrival. Therefore, it is (10) _____ to visit someone without having been invited. Gifts are not required, but are
appreciated. Flowers, incense, or tea may be proper gifts for the hosts. Hosts also appreciate a small gift for their children or
elderly parents.
Question 1. A. light B. slight C. lightly D. lighted
Question 2. A. possible B. capable C. probable D. likely
Question 3. A. continued B. chased C. followed D. taken
Question 4. A. each one B. one other C. the other one D. one another
Question 5. A. relation B. relationship C. relations D. relatives
Question 6. A. younger B. most young C. youngest D. young
Question 7. A. easy B. basic C. fundamentally D. elementary
Question 8. A. changed B. transferred C. reversed D. exchanged
Question 9. A. sense B. sensation C. sensitivity D. sensible
Question 10. A. inactive B. inaccurate C. inappropriate D. inexact

Read the following passage and mark the letter A, B, C or D on your answer sheet to indicate the correct answer to each of
the questions
Early peoples had no need of engineering works to supply their water. Hunters and nomads camped near natural
sources of fresh water, and populations were so sparse that pollution of the water supply was not a serious problem. After
community life developed and agricultural villages became urban centres, the problem of supplying water became important
for inhabitants of a city, as well as for irrigation of the farms surrounding the city. Irrigation works were known in prehistoric
times, and before 2000 BC the rulers of Babylonia and Egypt constructed systems of dams and canals to impound the flood
waters of the Euphrates and Nile rivers, controlling floods and providing irrigation water throughout the dry season. Such
irrigation canals also supplied water for domestic purposes. The first people to consider the sanitation of their water supply
were the ancient Romans, who constructed a vast system of aqueducts to bring the clean waters of the Apennine Mountains
into the city and built basins and filters along these mains to ensure the clarity of the water. The construction of such extensive
water-supply systems declined when the Roman Empire disintegrated, and for several centuries local springs and wells formed
the main source of domestic and industrial water.
The invention of the force pump in England in the middle of the 16th century greatly extended the possibilities of
development of water-supply systems. In London, the first pumping waterworks was completed in 1562; it pumped river water
to a reservoir about 37 m above the level of the River Thames and from the reservoir the water was distributed by gravity,
through lead pipes, to buildings in the vicinity.
Increased per-capita demand has coincided with water shortages in many countries. Southeast England, for example,
receives only 14 per cent of Britain's rainfall, has 30 per cent of its population, and has experienced declining winter rainfall
since the 1980s.
In recent years a great deal of interest has been shown in the conversion of seawater to fresh water to provide drinking
water for very dry areas, such as the Middle East. Several different processes, including distillation, electrodialysis, reverse
osmosis, and direct-freeze evaporation, have been developed for this purpose. Some of these processes have been used in large
facilities in the United States. Although these processes are successful, the cost of treating seawater is much higher than that
for treating fresh water.
From A. Briggs’ article on culture, Microsoft® Student 2008
Question 1: Early peoples didn’t need water supply engineering works because ______.
A. their community life had already developed
B. natural sources of fresh water nearby were always available
C. there was almost no dry season in prehistoric times
D. they had good ways to irrigate their farms
Question 2: The word “impound” in paragraph 1 is closest in meaning to “______”.
A. supply B. irrigate C. provide D. drain
Question 3: Clean water supply was first taken into consideration by ______.
A. the English people B. the ancient Romans C. the Egyptians D. the US people .
Question 4: Which of the following is NOT mentioned as a process of conversing seawater to freshwater?
A. Steaming and cooling. B. Water evaporation.
C. Dissolving chemicals. D. Purification method.
Question 5: In the passage, the author mainly discusses ______.
A. the development of water supply B. the results of water shortages
C. the water pumping systems D. the fresh water storage

Read the passage and choose the best answers to questions below. (10 pts)
In today’s culture, tattoos are very popular as a form of body decoration. As seen in their prevalence among
rebellious teenagers, social outsiders, dangerous outlaws, and many other people, tattoos are viewed as luxurious
fashion statements that possess a forbidden attraction -mainly as long as they remain unseen . However, some people get
tattoos in order to identify themselves with certain groups, such as military units, fraternities, or even criminal organizations,
like the Japanese yakuza. Tattoos have been used in this manner throughout history by many different cultures in the world. In
fact, they have always been an important functional and symbolic feature in most tribal and ancient cultures, whether they were
used for identification, worship, therapy, decoration, or in other rituals.
The earliest written records describing tattoo usage date back to Ancient Egypt, from where the practice spread to
Crete, Greece, Persia, and Arabia. However, tribal cultures throughout northern Europe, the western hemisphere, and east Asia
had been applying tattoos for several thousands of years before this, with people in Japan reportedly creating tattoos ten
thousand years ago. One of the best pieces of evidence of this usage came from the excavation of Otzi the Ice Man, the Oldest
naturally preserved body discovered in Europe. Dating back to 3300BC, this mummy had fifty-seven tattoos on its body,
mostly located on the inside of the left knee, ankle, and legs. In China and Russia, naturally mummified bodies that are just as
old have been found to bear tattoos in similar sports. Based on the location of these tattoos, some researchers speculate that
they were part of a therapeutic treatment for bones.
Ancient and tribal cultures used tattoos primarily for identification, though. A person’s tattoo would indicate their tribe,
family, or social status. For instance, the Norse, the Danes, the Saxons, and other Germanic peoples used tattoos to identify
their particular clans. The Maori in New Zealand used tattoos to indicate their social ranking. In many southeast Asian tribes,
as well as in American tribes like the Inuit, women used tattoos to indicate that they were eligible for marriage or already
married. Tattoos were also very common among warriors who displayed bravery or ferocity in combat. Chickasaw warriors
distinguished themselves with tattoos, and the Dayak tribes used them to boast of the people they killed in battle. In later
military usage, ancient Greek spies had tattoos that related their military ranks to each other. Originally, Roman soldiers did
not have tattoos. However, after witnessing the fierceness of British tribes they fought, Roman soldiers began to get tattoos in
order to emulate their enemy’s ferocity.
1. Which of the sentence below best expresses the essential information in the highlighted sentence in the passage?
A. Tattoos are popular with rebellious people who like to remain unseen, like teenagers, outlaws, and outsiders.
B. Tattoos have a rebellious attraction for many different kinds of people, but they usually must stay covered up.
C. Tattoos are attractive to different kinds of people who want to display their rebellious instincts for everyone.
D. Tattoos are such luxurious fashion statements that only a small number of people can get them, so they’re rarely seen.
2. In paragraph 1, the author mentions the modern use of tattoos as a form of identification in order to
A. introduce their origins in older cultures B. explain their associations with criminals
C. provide technical details on tattoos D. condemn them as fashion statements
3. The word "fraternities" in paragraph 1 is closest in meaning to
A. markets B. universities C. societies D. stores
4. According to paragraph 2, some researchers believe that the tattoos found on mummies were used to
A. mark them as slaves B. treat bone ailments
C. begin their adulthood D. assist them in worship
5. Based on the information in paragraph 2, what can be inferred about the origins of tattoos?
A. All ancient cultures copied the practice from Egypt.
B. Otzi the Ice Man is the first to have ancient tattoos.
C. It took thousands of years for tattoos to be used globally.
D. Tattoos developed independently in different cultures.
6. The word “them” in paragraph 3 refers to
A. warriors B. themselves C. tattoos D. tribes
7. The word "eligible" in paragraph 3 could best be replaced by
A. intelligent B. young enough C. trained D. qualified
8. According to paragraph 3, why did the Roman soldiers begin to get tattoos?
A. They were inspired by the ferocity of their tattooed enemies.
B. They wanted to boast of the enemies they killed.
C. They needed to identify their military ranks.
D. They used tattoos to treat their battle wounds.
9. According to paragraph 3, all of the following are listed as things identified by tattoos in tribal cultures EXCEPT
A. marital status B. tribal affiliation C. social rank D. criminal status
10. Which of the following can be the best title of the passage?
A. Tattoos – a rebellious attraction B. Tattoos on ancient mummies
C. Usage of tattoos as identification D. Tattoos in tribal and ancient cultures
I. Find the word in each group whose bold part is pronounced differently. (2.5 pts)
Write your answer (A, B, C or D) in the space provided on the right.
Câu 1. A. education B. question C. suggestion D. digestion -----------
Câu 2. A. school B. scheme C. schwa D. scholar -----------
Câu 3. A. swallow B. switch C. sweet D. sword -----------
Câu 4. A. booked B. escaped C. weighed D. laughed -----------
Câu 5. A. pays B. days C. says D. ways -----------
II. Choose the word with the different stress pattern in each group. (2.5 pts)
Write your answer (A, B, C or D) in the space provided on the right.

Câu A. convention B. restaurant C. interest D. document ----------


1
Câu 2. A. harmfulness B .refreshment C. prevention D. rotation ---------
Câu 3. A. together B. occasion C. forget D. favourite -----------
Câu 4 A.disappearance B. scientist C. conservation D. generation ----------
Câu 5 A.acquaintaince B. economic C. international D. satisfaction --------- PART II:
VOCABULARY AND GRAMMAR
I. Choose the word or phrase to complete each sentence.
Write your answer (A, B, C or D) in the space provided on the right. (5 pts)
1. According to the ____ of the contact, tenants must give six months’ notice if they intend to leave.
A. laws B. rules C. terms D. Details ______
2. The injured man was taken to hospital and _____ for internal injuries.
A. cured B. healed C. operated D. treated ______
3. The door hinges had all been oiled to stop them _____.
A. squeaking B. screeching C. shrieking D. squealin _______
4. Mary attempted to _____ herself with her new boss by volunteering to take on extra work.
A. gratify B. please C. ingratiate D. commend ________
5. You are under no obligation ______ to accept this offer.
A. indeed B. eventually C. apart D. whatsoever _____
6. In spite of his poor education, he was a most ______ speaker.
A. articulate B. ambiguous C. attentive D. authoritarian______
7. Wasn’t it you yourself ______ the door open?
A. to leave B. to have left C. who left D. that should leave _________
8. You should ______ at least three days for the journey.
A. expect B. permit C. accept D. allow ______
9. _____ of the financial crisis, all they could do was hold on and hope that things would improve.
A. At the bottom B. At the height C. On the top D. In the end __________
10. Could you possibly _____ me at the next committee meeting?
A. stand in for B. make up for C. fall back on D. keep in with ______

Đề 21
VOCABULARY AND GRAMMAR
II.Choose the best word or phrase to complete each sentence.
Write your answers (A, B,C or D) on the answer sheet. (5 pts)

1. Winning the cup in 1998 was just a ___ in the pan – they haven’t won the match since then.
A. blaze B. flame C. light D. flash
2. When Tom ran out of money, he _____ his sister for help.
A. fell in with B. fell upon C. fell behind D. fell back on
3. Of the two films we watched last night, “The Avengers” is ______.
A. more interesting B. as interesting C. the more interesting D. the most interesting
4. Some parts of Asia are extremely crowded. _____, there are many forests where few people live.
A. Moreover B. Nevertheless C. Consequently D. Otherwise
5. Jessica: “______”
Mark: “No, of course not. Get some rest, and I will see you tomorrow.”
A. Would you mind if I went home? I am feeling really nauseous.
B. Did you buy me any sleeping pills on your way home?
C. May I take a nap before we leave for the concert tonight?
D. Would you mind turning down the volume? It‘s far too loud for this time of night.
6. Your new heating system saves money but your rates are increased because you’ve improved your home. It’s a(n) ______
circle.
A. vicious B. cruel C. ugly D. nasty
7. The football match tomorrow evening will be broadcast _______on TV and radio.
A. simultaneously B. communally C. uniformly D. jointly
8. My husband always makes children hit the______before 9pm.
A. roof B. ceiling C. bed D. hay
9.I can’t afford to buy a new coat this winter. I’ll have to ________ the one I have.
A. make light with B. make myself at home with
C. make amends D. make do with
10.Her father is a _______ drinker. He is always drinking a lot of wine before going to bed.
A. strong B. heavy C. addictive D. capable

I. Choose the word whose underlined part is pronounced differently from that of the others.
Write your answers (A, B, C or D) on the answer sheet. (2.5 pts)
1. A. considerate B. candidate C. associate D. adequate
2. A. geyser B. honeymoon C. journey D. money
3. A. comb B. abstract C. subtle D. doubt
4. A. releases B. misuses C. increases D. collapses
5. A. removed B. approved C. beloved D. relieved

Write your answers (A, B, C or D) on the answer sheet. (2.5 pts)


1. A. investigate B. investment C. indicate D. immediate
2. A. workforce B. career C. downside D. battle
3. A. argumentative B. psychological C. contributory D. hypersensitive
4. A. secretary B. legendary C. directory D. dormitory
5. A. farewell B. milestone C. headline D. windscreen

I. Read the following passage and choose the correct word to fill each gap.
Write your answer in the numbered blanks that follow. (5 pts)
If you’re an environmentalist, plastic is a word you tend to say with a sneer or a snarl. It has become a symbol of our
wasteful, throw-way society. But there seems little (1)______ it is here to stay, and the truth is, of course, that plastic has
brought enormous (2)______ even environmental ones. It’s not really the plastics themselves that are the environmental
(3)______ – it’s the way society choose to use and abuse them.
Almost all the 50 or so different kinds of modern plastic are made from oil, gas or coal – non-renewable natural (4)______.
We import well over three million tons of the stuff in Britain each year and, sooner or later, most of it is thrown away. A high
proportion of our annual consumption is in the (5)______ of packaging, and this (6)______ about seven per cent by weight of
our domestic refuse. Almost all of it could be recycled, but very little of it is, though the plastic recycling (7)______ is growing
fast.
The plastics themselves are extremely energy-rich – they have a higher calorific (8)______ than coal and one (9)______ of
‘recovery’ strongly favored by the plastic manufacturers is the (10)______ of waste plastic into a fuel.
1. A. evidence B. concern C. doubt D. likelihood
2. A. pleasures B. benefits C. savings D. profits
3. A. poison B. disaster C. disadvantage D. evil
4. A. resources B. processes C. products D. fuels
5. A. way B. kind C form D. type
6. A. takes B. makes C. carries D. constitutes
7. A. manufacture B. plant C. factory D. industry
8. A. degree B. value C. demand D. effect
9. A. measure B. mechanism C. method D. medium
10. A. conversion B. melting C. change D. replacement
II. Read the passage carefully, then choose the correct option (marked A, B, C or D) to answer the questions.
Write your answer in the numbered blanks that follow. ( 5 pts )
Scientists have established that influenza viruses taken from man can cause disease in animals. In addition, man can
catch the disease from animals. In fact, a greater numbers of wild birds seem to carry the virus without showing any evidences
of illness. Some scientists conclude that a large family of influenza virus may have evolved in the bird kingdom, a group that
has been on earth 100 million years and is able to carry the virus without contracting the disease. There is even convincing
evidence to show that virus strain are transmitted from place to place and from continent to continent by migrating birds.
It is known that two influenza viruses can recombine when both are present in an animal at the same time. The result
of such recombination is a great variety of strains containing different H and N spikes. This raises the possibility that a human
influenza virus can recombine with an influenza virus from a lower animal to produce an entirely new spike. Research is
underway to determine if that is the way major new strains come into being. Another possibility is that two animal influenza
strains may recombine in a pig, for example, to produce a new strain which is transmitted to man.
1. According to the passage, scientists have discovered that influenza viruses ______.
A. cause ill health in wild animals B. do not always cause symptoms in birds
C. are rarely present in wild birds D. change when transmitted from animals to man
2. What is known about the influenza virus?
A. It was first found in a group of very old birds. B. All the different strains can be found in wild birds.
C. It existed over 100 million years ago. D. It can survive in many different places.
3. According to the passage, a great variety of influenza strains can appear when______.
A. H and N spikes are produced B. animal and bird viruses are combined
C. dissimilar types of viruses recombine D. two viruses of the same type are contracted
4. New strains of viruses are transmitted to man by_______.
A. a type of wild pig B. diseased lower animals
C. a group of migrating birds D. a variety of means
5. It can be inferred from the passage that all of the following are ways of producing new strains of influenza
EXCEPT_______.
A. two influenza viruses in the same animal recombining B. animal viruses recombining with human viruses
C. two animal viruses recombining D. two animal viruses
recombining in a human

III.Read the following passage and choose the correct answer to each question.
Write your answers on the answer sheet. (10 pts)
The Nobel Peace Prize is awarded annually and the first woman to win this prize was Baroness Bertha Felicie Sophie von
Suttner in 1905. In fact, her work inspired the creation of the Prize. The first American woman to win this prize was Jane
Addams, in 1931. However, Addams is best known as the founder of Hull House.
Jane Addams was born in 1860, into a wealthy family. She was one of a small number of women in her generation to graduate
from college. Her commitment to improving the lives of those around her led her to work for social reform and world peace.
In the 1880s Jane Addams traveled to Europe. While she was in London, she visited a ‘settlement house’ called Toynbee Hall.
Inspired by Toynbee Hall, Addams and her friend, Ellen Gates Starr, opened Hull House in a neighborhood of slums in
Chicago in 1899. Hull House provided a day care center for children of working mothers, a community kitchen, and visiting
nurses. Addams and her staff gave classes in English literacy, art, and other subjects. Hull House also became a meeting place
for clubs and labor unions. Most of the people who worked with Addams in Hull House were well educated, middle-class
women. Hull House gave them an opportunity to use their education and it provided a training ground for careers in social
work.
Before World War I, Addams was probably the most beloved woman in America. In a newspaper poll that asked, “Who among
our contemporaries are of the most value to the community?” Jane Addams was rated second, after Thomas Edison. When
she opposed America’s involvement in World War I, however, newspaper editors called her a traitor and a fool, but she never
changed her mind. Jane Addams was a strong champion of several other causes. Until 1920, American women could not vote.
Addams joined in the movement for women’s suffrage and was a vice president of the National American Woman Suffrage
Association. She was a founding member of the National Association for the Advancement of Colored People (NAACP), and
was president of the Women’s International League for Peace and Freedom. Her reputation was gradually restored during the
last years of her life. She died of cancer in 1935.
Question 1: With which of the following subjects is the passage mainly concerned?
A. The first award of the Nobel Peace Prize to an American woman
B. A woman’s work for social reform and world peace
C. The early development of Social Work in America
D. Contributions of educated women to American society
Question 2: Which of the following can be inferred from the passage?
A. the work of Baroness Bertha Felicie Sophie von Suttner was an inspiration to Jane Addams.
B. Jane Addams is most famous for her opening of Hull House.
C. those who lived near Hull House had very poor literacy skills.
D. Jane Addams considered herself as a citizen of the world rather than of one particular country.
Question 3:The word “commitment” in line 5 is closest in meaning to ______.
A. involvement                       B. obligation                 C. dedication                D. enthusiasm
Question 4: Jane Addams was inspired to open Hull House because ______.
A. it gave educated women an opportunity to use their education and develop careers in social work
B. she traveled to Europe in the 1880s
C. she visited Toynbee Hall
D. she was invited by a ‘settlement house’ in Chicago
Question 5:The word “their” in line 12 refers to ______.
A. children of working mothers B. middle-class women
C. visiting nurses                                                     D. labor union members
Question 6: The word “it” in line 12 refers to ______.
A. opportunity                    B. Hull House             C. meeting place           D. Toynbee Hall
Question 7: The word “contemporaries” in line 14 is closest in meaning to ______.
A.  people of the same time                                    B. famous people still alive
C. elected officials                                                   D. people old enough to vote
Question 8: According to the passage, Jane Addams’ reputation was damaged when she ______.
A. allowed Hull House to become a meeting place for clubs and labor unions
B. joined in the movement for women’s suffrage
C. became a founding member of the NAACP
D. opposed America’s involvement in World War I
Question 9: Where in the passage does the author mention the services provided by Hull House?
A. Lines 1-3                      B. Lines 8-12              C. Lines 13-16          D. Lines 17-21
Question 10: The word “restored” in line 20 is closest in meaning to ______.
A. brought back                B. improved                C. changed                  D. bettered

II. Rewrite each of the sentences below in such a way that its meaning stays the same, using the wordgiven in capitals. Do
NOT change this word in any way.(5 pts)
1. The only way you can become a good English speaker is by practising it every day. (ONLY BY)
…………………………………………………………………………………………………………...
2. I am sure you had a good time at the party. (MUST)
…………………………………………………………………………………………………………...
3. Peter didn’t feel like going to the party. (MOOD)
…………………………………………………………………………………………………………...
4. I could hold a big party because of Mom’s help. (HAD)
…………………………………………………………………………………………………………...
5. “You damaged my bicycle, John!” said Margaret. (ACCUSED)
…………………………………………………………………………………………………………...

I. Finish each of the sentences below in such a way that it means the same as the original one.(10 pts)
1. She wore headphones in order not to disturb other classmates.
 She wore headphones lest________________________________________________.
2. Success in the academic field depends on your ability to amass qualification.
The more _____________________________________________________________.
3. That reminds me of the time I climbed to the top of Mount Fuji.
 That takes me___________________________________________________________
4. The machine is undeniably of great help .
 It is __________________________________________________________________.
5. We weren't surprised by his success.
 His success came ________________________________________________________
6. More butter is eaten in America than in Europe.
 Not so_________________________________________________________________
7. The staff hated Frank’s new policies so intensely that they went on strike.
 So intense _____________________________________________________________
8. He said that he had won as a result of good luck.
 He attributed___________________________________________________________
9.We cannot see animals in a vast area after the forest fire.
There is an_____________________________________________________________
10.They listened eagerly and attentively to the president's speech.
 They were ____________________________________________________________

III. Write an essay of about 200 words about the topic “ The only way to improve safety of our road is to give much
stricter punishments on driving offences. To what extend do you agree or disagree?
III. Read the following text which contains 10 mistakes. Identify the errors and write the corrections in the corresponding
numbered boxes. (10 pts)
Example: Line 1: ever => never
Line
1 The market for tourism in remote areas is booming as ever before. Countries all across the world are active
2 promoting their 'wilderness' regions – such as mountains, Arctic lands, deserts, small islands and wetlands – to highly
3 spending tourists. The attraction of these areas is obvious: by defining, wilderness tourism requires little or no initial
4 investment. But that does not mean that there is no cost. Like the 1992 UN Conference on Environment and
5 Development recognized, these regions are fragile (i.e. highly vulnerable of abnormal pressures) not just in terms of
6 the culture of their inhabitation. The three most significant types of fragile environment in these respects are deserts,
7 mountains and Arctic areas. An important character is their marked seasonality. Consequently, most human acts,
8 including tourism, are limited to clearly defined parts of the year.
9 Tourists are drawn to these regions by their natural beauty and the unique culture of its people. And poor
10 governments in these areas have welcomed the 'adventure tourists', grateful for the currency they bring. For several
11 years, tourism is the prime source of foreign exchange in Nepal and Bhutan. Tourism is also a key element in the
12 economics of Arctic zones such as Lapland and Alaska and in desert areas such as Ayres Rocks in Australia and
13 Arizona’s Monument Valley.

II. Fill in each gap with a suitable phrasal verb in the correct form to complete the sentences.
Write your answers on the answer sheet. (10 pts)
live on look through pass down set off turn down
close down come back deal with get up keep up with
1. They offered her a trip to Europe but she _________it .
2. He has the report and found nothing interesting.
3. Phong walks too fast and it’s really hard to him.
4. These traditional stories have been from parents to children over many generations.
5. My sister is an early bird. She at 7 o’clock every day of the week.
6. We’ll for Tay Ho village at 9 o’clock and arrive at 10 o’clock.
7. When I was a student, I small allowances from my parents.
8. The government must now the preservation of traditional craft villages.
9. The banks have a lot of branches in villages over the last few years.
10. Many foreign tourists decided to Viet Nam for another holiday
I. Supply the correct form of the words given in brackets to complete the sentences.
Write your answers on the answer sheet. (5 pts)

1. Jenny has sent me a very _____ (APOLOGY) letter explaining why she didn’t do what she promised.
2. People used to suffer from their life-time physical ________(NORMAL)
3. If you think I am going to marry you, then I’m afraid you’re very much ____ (TAKE).
4. The man was found guilty of fraud and sentenced to three years _____(PRISON )
5. He won the discus event at the Olympic Games but was later _____( QUALIFY ) when a medical check proved that he
had been taking drugs
6. The recent hurricane caused ___ ( WIDE) damaged
7. Jim is one of the most ___ (SPEAK ) members of the committee
8. We try to ensure the ____( BE) of our employees
9. The ________ (PAGE) of this document is wrong! There’s no page 13
10. It’s essential that the application forms be sent back before the .....(LINE)
Answers:
I. Fill each gap in the following passage with ONE suitable word. Write your answers on the answer sheet. (5 pts)
Health experts have warned that the current soaring obesity levels in European children may (1) ________ create
health problems of epidemic proportions within the next three decades. Over the past ten years, obesity in six-year olds has
doubled (2) ________ the number of obese fifteen-year olds has trebled. If present trends continue, by 2030 between 60 and
70 percent of Europeans will be overweight and 40 to 50 percent will be obese. The (3) ________ rise in weight-related
diabetes, high blood pressure, heart disease and some forms of cancer will (4) ________ intolerable strains on health services
unless steps are taken to address the problem.
The explosion in child obesity can be put (5) ________ to the increasingly inactive lifestyles of modern children. Many
parents, worried lest their children (6) ________ come to (7) ________ from traffic or strangers, discourage them (8)
________ playing outdoors and instead provide ever more opportunities for sedentary entertainment in the home. (9)
________ should the influence of the food industry be underestimated: (10) _____ spends one thousand times more on
advertising fast foods and convenience foods in Europe and America than the total budget for promoting health in the same
two regions.

Đề 22
Put each word in brackets into an appropriate form.
1. The government stressed it was not abandoning its tough __________ policies. (INFLATE)
2. The tsunami has caused __________ damage to the Japanese. (CALCULATE)
3. The woman detected her ________ conditions soon after she felt ill in her body. (CANCER)
4. All those foreign visitors can speak Vietnamese very well. There will be a big (USEFUL)
__________ to have a translator here.
5. I cast my eyes ________ and saw the geese flying in V-line. (HEAVEN)
6. ________ programs are offered to those who want to do professional training while (SERVICE)
being paid to work.
7. The behavior of the woman is very __________. The guard of the supermarket is having (SUSPECT)
a look on her.
8. It is inhumane to sell ________ limbs of the animals, such as rhinoceros or elephants, in (MEMBER)
the market place.
9. The TV serial presented a scene of happy __________ . (DOMESTIC)
10. He was regarded as a _______ winner as he beat his opponents in three straight sets. (RUN)
Complete the sentences below, using a phrasal verb from the box in its correct form.
bring out do up slow down save up hang over
mix up grow up go with dress up run for

1. Silence ___the theatre as the audience awaited the opening curtain with expectation and excitement.
2. Having seen a sharp bend ahead, Tim pressed hard on the brake pedal to ___.
3. Charles Dickens was born near Portsmouth, Hampshire on 7 February 1812, but relocated to and ___ in Camden Town in
London.
4. Barrack Obama has decided to ___ the American presidency in the election that will take place next year.
5. Because I hate ___ my shoes, I have bought a pair of shoes without any laces.
6. Sarah wanted to buy some curtains that would ___ his furniture, so she had brought a photo of her sofa with her to the store.
7. Kate had been ___ in order to buy a new laptop, but then she decided to use the money on the guitar lessons instead.
8. Even though two of the bands are dead, a new ‘The Beatle’ album called Love was recently ___.
9. Brendan was worried about having to ___ for the boss’ retirement dinner as he didn’t own any formal clothes.
10. He ___ his grandmother’s phone number with his girlfriend’s, which led to some embarrassment for him.
The following passage contains 10 mistakes. Identify the mistakes and write the corrections in the corresponding numbered
boxes on your answer sheet.
One area of paleoanthropological study involves the eating and dietary habits of hominids, erect bipedal primates
including early humans. It is clear that at some stages of history, humans began to carry their food to central places, calling
home bases, where it was shared and consumed by the young and other adults. The use of home bases is a fundamental
component of human society behavior; the common meal served at a common hearth is a powerful symbol, a mark of social
unity. Home base behavior does not occur
among nonhuman primates and is rare among mammals. It is unclear that humans began to use home bases, what kind of
communications and social relations are involved, and what the ecological and food-choice contexts of the shift were. Work
with early tools, surveys of paleoanthropological sites, development and test of broad ecological theories, and advances in
comparative primatology are contributing to knowledge about this central chapter in human prehistoric.
Line 1
Line 2
Line 3
Line 4
Line 5
Line 6
Line 7
Line 8
Line 9
Line 10
Line 11
Line 12 Line 13
Line 14
Line 15

Read the passage below and choose the correct word or phrase to complete each of the blanks. (5 pts)
It is because of the centrality of the city that the financial markets have (1)_______ put. It had been (2) _______ forecast that
they would move out en masse, financial work (3) _______ among the most quantitative and computerized of functions. A lot
of the back-office work has been relocated. The main business, (4) _______, is not record keeping and support services; it is
people sizing (5) _______ other people, and the center is the place for that.
The problems, of course, are immense. To be an optimist (6) _______ the city, one must believe that it will lurch from
crisis to crisis but somehow survive. Utopia is (7) _______ in sight and probably never will be. The city is (8) _______ mixed
up for that. Its strengths and its ills are inextricably bound (9) _______. The same concentration that makes the center efficient
is the cause of its crowding and the destruction of its sun and its light and its scale. Many of the city’s problems, (10) _______,
are external in origin – for example, the cruel demographics of peripheral growth, which are difficult enough to forecast, let
alone do
Finish each of the following sentences in such a way that it means the same as the original one(s).
1. We couldn’t have managed without my father’s money.
If it hadn’t.........................................................................................................
2. House prices have risen dramatically this year.
There has ........................................................................................................
3. The only thing they didn’t steal was the television.
They stole ....................................................................................................
4. He didn’t succeeded in searching for the stolen car.
He tried in vain ...... ............................................................................................................
5. She started to clean up the house just after the guests had left.
No sooner .........................................................................................................
6. The boy was about to cry when he was reprimanded by his mother.
→ The boy was on ...........................................................................................................
7. People believe that the Chinese invented paper in 105 A.D.
→ Paper............................................................................................................................
8. As people use a lot of wood-pulp, many trees are cut down.
→ The more ....................................................................................................................
9. It takes six hours to drive from here to London.
→ It is .................................................................................................................................
10. He got so angry that no one dared to say anything.
→ So ...................................................................................................................................
Rewrite the sentences

1. A new house is impossible – we can not afford it. QUESTION

2. The fact that the judge had been involved in some illegal activities was not revealed until several years after his death.
LIGHT

3. Don’t forget those who are at work on this lovely, sunny day! SPARE

4. The severity of the punishment bore no relation to the seriousness of the crime.
PROPORTION

5. The impression most people have of him is that he is an honest person. COMES

Every students should attend extra classes after school (free- paying). What is your opinion?
Your writing must be of between 150 - 200 words. Do not mention any personal information Mark the letter A, B, C, or D on
your answer sheet to indicate the word whose underlined part differs from the other three in pronunciation in the
following question.
1. A. dinosaur B. diorit C. diode D. dint
2. A. baths B. bathymetry C. clothier D. thaw
3. A. close B. close-up C. nosegay D. nosy
4. A. huddle B. hooch C. hoody D. hautboy
5. A. tear B. teasel C. tearful D. tear- gland
Mark the letter A, B, C, or D on your answer sheet to indicate the word that differs from the rest in the position of the main
stress in each of the following questions.
1. A. remote B. receive C. prevent D. recent
2. A. preferential B. congratulate C. development D. preservative
3. A. abnormal B. initial C. innocent D. impatient
4. A. interchange B. infamous C. overeat D. undercurrent
5. A. aphorism B. extremism C. barbarism D. feminism
CAUHOI
1. A number of oil tankers have been laid ________ recently.
A. on B. down C. up D. in
2. The inscription on the tombstone had been worn ________ by the weather and could scarcely be read.
A. away B. off C. out D. on
3. ________ second thoughts, I'd rather not go out tonight.
A. With B. In C. On D. Under
4. The woman accused of shoplifting was found not guilty and was ________.
A. excused B. liberated C. acquitted D. interned
5. My car was so old that I could only sell it for ________.
A. rubbish B. scrap C. debris D. waste
6. Not only ________ in the project, but he also wanted to become the leader.
A. did Jack involve B. had Jack been involved
C. was Jack involved D. Jack was involved
7. I felt a bit ________ and seemed to have more aches and pains than usual.
A. out of sorts B. on the mend C. over the worst D. under the fevers
8. The school was closed for a month because of a serious ________ of fever.
A. outcome B. outburst C. outset D. outbreak
9. I don't think he's ever been there, ________?
A. do I B. has he C. have I D. hasn't he
10. As a citizen, it is normal to ________ to the laws and rules made by the society.
A. conform B. resist C. hinder D. obey
Complete the following passage by choosing the correct option (A, B, C or D) to fill in blanks. (5pts)
The Alexandra Palace in north London was built with private funds as a “People’s Palace”. Serviced by its own
station, it was opened in 1873 and was extremely well (1)_______ until, two weeks after its opening, it burnt down. It was
replaced by a slightly larger building which opened in 1875 and featured, (2)________other things, a splendid organ an Great
Hall, which was the size of a football pitch. Despite the extraordinarily wide range of events (3)_______ there – from dog
shows to great concerts and banquets, from elephant displays to bicycle matches – it always operated at a loss and by 1877
much of the park around it had been sold to speculative builders, leaving only about half of the original land.
In 1900, a committee was appointed, whose principal duty was to run the palace and park “for the free use of the
people forever.” There were, however, (4)________ to charge for entry so that the substantial costs could be (5)_______ . The
Palace continued, with (6)________ degrees of success, as an entertainment centre. In the 1930s, it was probably most
(7)________ for being the home of the world’s first high definition television broadcasts.
In 1980 the building was once more devastated by fire and (8)_______ to a ruin. It was then decided to (9)_________
it and to create a major exhibition centre with community (10)_______, such as a restaurant and a health club.
1. A. inhabited B. attended C. crowded D. visited
2. A. among B. between C. from D. around
3. A. performed B. set C. staged D. laid
4. A. powers B. terms C. allowances D. authorities
5. A. fulfilled B. covered C. matched D. made
6. A. unsteady B. varying C. altering D. unsettled
7. A. distinct B. marked C. considerable D. notable
8. A. turned B. converted C. reduced D. wrecked
9. A. recover B. revise C. restore D. reform
10. A. facilities B. conveniences C. supplies D. appliances
Read the following passage and choose the option that indicates the correct answer to each of the following questions. (5
pts)
E-books in future
Over the past decade, e-books have revolutionized the way people consume written formation. It will probably come as no
surprise the total global sales number continues to rise each year. There are several reasons why readers are buying more e-
books. For starters, reading a lengths e-book in front of your computer isn't the most comfortable thing to do. But since tablet
sales have exploded, readers can now relax and read their favourite e-books in bed from the beach. Many e-books are often
sold at lower prices than traditional books. Since there is almost zero cost for producing e-books, some authors are offering
their novels at lower prices. Another factor is convenience. In the past, keeping a large book collection meant setting aside a lot
of extra space in one's home. With e-books, it is now possible for readers to carry a portable library, which contains thousands
of books. To really understand e-book market though, it is important to keep up with national trends.
Recent data suggests that the e-book market in the US and the UK have matured. In both of these countries, e-book
sales account for roughly 20 percent of overall book sales. Although that percentage continues to increase, it is going up at a
slower rate than in the past.
The popularity of e-books has exploded in Russia. In one poll, 70 percent of Russians claim to have read at least one e-book.
However, publishers are not too pleased with this new because approximately 92 percent of e-books in Russia are acquired
illegally.
In France, consumers don't appear to be loo keen on e-books at this time. Although in 2012, e-book sales comprised about 3
percent of the market, it is predicted that this number will increase at a very slow pace. Some French people believe that
reading e-books on small screens is uncomfortable. Others say that the French are a cultural exception, as they like the feeling
of holding a dusty old physical book. This demonstrates that no matter how popular e- books get, it is unlikely that traditional
books will disappear any time soon.
Câu 1. This year, e-book sales in the US and the UK will probably ____
A. stay the same B. start to decline
C. increase drastically D. keep going up
Câu 2. Which of the following is NOT a reason why e-books became popular worldwide?
A. People can now store more books easily.B. Most popular novels are only offered as e-books.
C. Reading e-books recently became more comfortable.D. A new type of reading device was invented.
Câu 3. The phrase “account for" in paragraph 2 is closest in meaning to ____
A. form the total of B. are the explanation of
C. are a particular part of D. know what has happened
Câu 4. All of the following are the reasons why the French are not fond of e-books very much EXCEPT that ___.
A. the French may want to keep their traditional reading habit
B. it is considered that reading e-books on small screens is not comfortable
C. the French have a trend of reading traditional books
D. the French are an exception, not using any mobile devices.
Câu 5. What is the meaning of the last sentence of the passage?
A. French people will change their minds about e-books.
B. Some people will continue to read traditional books.
C. It is just a matter of time before e-books disappear.
D. E-books won’t get popular in France until screens get bigger.
Part 3: Read the following passage and choose the correct answer to each question.
Write your answers on the answer sheet. (10 pts)
The body of an adult insect is subdivided into three sections, including a head, a three-segment thorax, and segmented
abdomen. Ordinarily, the thorax bears three pairs of legs and a single or double pair of wings. The vision of most adult insects
is specialized through two large compound eyes and multiple simple eyes.
Features of an insect’s mouth parts are used in classifying insects into types. Biting mouth parts, called mandibles, such as the
mouth parts found in grasshoppers and beetles, are common among insects. Behind the mandibles are located the maxillae, or
lower jaw parts, which serve to direct food into the mouth between the jaws. A labrum above and one below are similar to
another animal’s upper and lower lips. In an insect with a sucking mouth function, the mandibles, maxillae, labrum, and
labium are modified in such a way that they constitute a tube through which liquid such as water, blood, or flower nectar can
be drawn. In a butterfly or moth, this coiled drinking tube is called the proboscis because of its resemblance, in miniature, to
the trunk of an elephant or a very large nose. Composed chiefly of modified maxillae fitted together, the insect’s proboscis can
be flexed and extended to reach nectar deep in a flower. In mosquitoes or aphids, mandibles and maxillae are modified to sharp
stylets with which the insect can drill through surfaces like human or vegetable skin membranes to reach juice. In a housefly,
the expanding labium forms a sponge like mouth pad that it can use to stamp over the surface of food, sopping up food
particles and juices.
Insects, the most numerous creatures on our planet, are also them most adaptable. They require little food because they are
small. They easily find shelter and protection in small crevices in trees and surface geological formations. Species of insects
can evolve quickly because of their rapid reproduction cycle; they live in every climate, some making their homes in the frozen
Arctic regions and many others choosing the humid, warm, and nutrient-rich rain forest environment. An active part of the
natural food cycle, insects provide nutrition for animals and devour waste products of other life forms.
1. What is the best title for this passage?
A. An Insect’s Environment B. The Structure of an Insect
C. Grasshoppers and Beetle D. The Stages of Life of an Insect
2. What is the purpose of this passage?
A. To complain B. To entertain C. To persuade  D. To inform
3. How are insects classified?
A. By the environment in which they live B. By the food they eat
C. By the structure of the mouth D. By the number and type of wings
4. The word common in paragraph 2 is closest in meaning to
A. normal  B. important C. rare D. necessary
5. What is the purpose of maxillae?
A. to bite or sting B. to drill through surfaces to find nourishment
C. to put food between the jaws D. to soak up nourishment like a sponge
6. The author compares labrum and labium to
A. an upper and lower lip  B. maxillae C. mandibles D. jaws
7. What is the proboscis?
A. Nectar B. A tube constructed of modified maxillae
C. A kind of butterfly D. A kind of flower
8. Which of the following have mandibles and maxillae that have been modified to sharp styles?
A. Grasshoppers  B. Mosquitoes C. Butterflies D. Houseflies
9. The phrase drill through in paragraph 2 could best be replaced by
A. penetrate  B. explore C. saturate D. distinguish
10. The word it in paragraph 2 refers to
A. pad B. housefly C. food  D. mouth

Đề 23
Put each word in brackets into an appropriate form.
1. Many people make their living on theater. In essence the theater is not only for (LIVE)
entertainment, it is a _____ . (ATTEND)
2. They never dare to leave their only child ________ for even a moment. (INDIVIDUAL)
3. What I don’t like about school uniform is that it completely destroys all_______. (IMPRESS)
4. The main reason I believe children shouldn’t be exposed to violence on TV is that they are (ESTIMATE)
so__________ at that age. (SYSTEM)
5. His ________ of his opponent led to his biggest failure. (BREAD)
6. The curriculum is ________developed; therefore, students benefit a great deal. (BENEFIT)
7. The mother is usually the homemaker and the father is the _______. (DESTROY)
8. There was a donation of $100,000 made by an anonymous _________. (COMPROMISE)
9. Being the hardest natural substance, diamond is practically_________.
10. The government’s ________ approach has brought criticism.
Complete the sentences below, using a phrasal verb from the box in its correct form.
on good terms independent of tired of out of tune
ashamed of on the whole capable of at most
for safe keeping out of order for a change
1. She was .................................... her poor exam results.
2. He gave his watch and wallet to me .................. while he went for a swim in the lake.
3. You are ........................................ better work than this.
4. The piano is ........................................ again. It sounds terrible.
5. She is old enough to be ........................................ her parents.
6. Despite many disagreements, they’re now ..........................with their next-door neighbors.
7. I’m ..........................................doing the same thing every day.
8. There weren’t many passengers on the bus – twelve ............................................
9. Parts of the play were boring, but ......................................... I quite enjoyed.
10. You’ll have to use the stairs. The lift is ...........................................
There are 10 errors in the following passage. Identify and correct them. The first has been done for you.
Most children with (0) health appetites are ready to eat almost anything that is offering them and a child rare dislikes food
unless it is badly cooked. The way the meal is cooked and served is most important and an attractive served meal will often
improve a child’s appetite. Never ask a child whether he likes and dislikes a food and never discuss them in front of him or
allow nobody else to do so. If the father says he hates fat meat or the mother refuses vegetables under the child’s hearing he is
likely to copy this procedure. Take it for grant that he likes everything and he probably will. Nothing healthful should be
omitted out the meal because of a supposing dislike. At meal time, it is a good idea to give a child a small portion and let him
come back for a second helping other than give him as much as he is likely to at all once. Do not talk too much to the child
during the meal times. But let him get on with his food, and do not allow him to leave the table immediately after a meal or he
will soon learn to swallow his food so that he can hurry back to his toys. Under no circumstance must a child be coaxed and
forced to eat.
Line 0
Line 1
Line 2
Line 3
Line 4
Line 5
Line 6
Line 7
Line 8
Line 9
Line 10
Line 11
Line 12
Line 13
Line 14
Line 15
Read the passage below and choose the correct word or phrase to complete each of the blanks. (5 pts)
Urbanization has been a constant (1) _______ in human development for the past ten thousand years. (2) _______ there has
always been the necessity for a minimum - sometimes a very substantial minimum - of the (3) _______ to work the land, such
surplus individuals as can be sustained have tended to gather in cities. Two thousand years ago, the cities of antiquity (4)
_______ an administrative center for the surrounding area, and a refuge from attack. In the Middle Ages, it was the city folk
(5) _______ were the first to break the stultifying grip of feudalism, and it was in the cities of Northern Italy that the
phenomenon of the Renaissance was (6) _______.
Today only a fraction of mankind lives in the country, and the (7) _______ which does so is decreasing. Mega-cities with
populations exceeding ten million are common, and they will become even (8) _______ in the coming decades. While many
(9) _______ of city life are unattractive - pollution, stress, and separation from (10) _______ to name but a few - more and
more of humanity seems to be choosing an urban existence. Adjusting to this fact is going to be one of the major challenges of
the coming century. And what of the country? The chances are that its relative decline will be reversed as the information
revolution continues, it is possible to enjoy the benefits of social contact without needing to leave one's rural residence.
Finish each of the following sentences in such a way that it means the same as the original one(s).
1. Return the product to the shop if you have any complaints about it.
→ Should
2. It’s almost nine months since I stopped subscribing to that magazine.
→ I cancelled
3. Her success went beyond her expectation.
→ Never
4. His fondness for the game increased with his proficiency.
→ The more
5. Simon hadn’t expected that he would feel so weak after the operation.
→ The operation left
6. I am sure he recognized us at the airport.
→ He must
7. The journalists only heard about the changes to the wedding plans when they arrived at the avenue.
→ Not until
8. “Why can’t you do your work more carefully?” Helen’s boss said to her.
→ Helen’s boss criticized
9. Someone has suggested raising the parking fees in the city.
→ It
10. Don’t misquote. I never said I hated ballet.
→ Don’t put
Rewrite the sentences
1. The manager told his staff that he was pleased, but he could do better. ROOM

2. They’ll cause a lot of trouble. CAT

3. Some people say that there’s hardly any difference between love and hate. LINE

4. She can buy anything she wants – She’s very rich. ROLLING

5. A rise in temperature in the next century seems likely. CHANCE

Some people think that family is the most important influence on young adults. Other people think that friends are the most
important influence on young adults. Which view do you agree with? Use example to support your opinion. ( You should write
at least 150 words)
Mark the letter A, B, C, or D on your answer sheet to indicate the word whose underlined part differs from the other
three in pronunciation in the following question.
1. A allegedly B confusedly C supposedly D wickedly
2. A youths B maps C cigarettes D months
3. A equation B television C mention D decision
4. A rise B rinse C browse D bruise
5. A substitute B muddled C shutter D substantial
Mark the letter A, B, C, or D on your answer sheet to indicate the word that differs from the rest in the position of the main
stress in each of the following questions.
1. A. attend B. appear C. anthem D. apply
2. A. potential B. understand C. recommend D. volunteer
3. A. catastrophe B. agriculture C. dictionary D. supervisor
4. A. diplomacy B. delicacy C. peninsula D. pneumonia
5. A. mimosa B. karate C. graffiti D. anecdote
CAUHOI
1. He attempted to ________ the painting from the gallery but he was caught.
A. rob B. steal C. thief D. kidnap
2. A: "Will the Jackson be invited?"- B:" I ________. "
A. expect so not B. so expect C. expect so D. don't expect
3. I read the contract again and again ________ avoiding making spelling mistakes.
A. with a view to B. on account of C. by means of D. in terms of
4. In a report submitted to the government yesterday, scientists ________ that the building of the bridge be stopped.
A. banned B. complained C. said D. recommended
5. I enjoyed reading that story. It was rather sad, ________.
A. also B. though C. but D. so
6. "I'm sure the Whitleys were involved." "They ________ have since they know nothing about the business".
A. can't B. wouldn't C. shouldn't D. mustn't
7. ________, the balcony chairs will be ruined in this weather.
A. Leaving uncovered B. Having left uncovered
C. Left uncovered D. Been left uncovered
8. His friends offered to ________ the next time he was in town, so that he wouldn't have to pay for a hotel.
A. place him in B. put him up C. back him up D. turn him out
9. Susan became so tired of city life that she decided to buy a piece of land ________
A. out of the ordinary B. as the crow flies
C. far and away D. in the middle of nowhere
10. The ________ of our cruise includes stops at six ports in three different countries.
A. timetable B. intention C. itinerary D. itemization
Complete the following passage by choosing the correct option (A, B, C or D) to fill in blanks. (5pts)
Everyone wants to reduce pollution. But the pollution problem is (1)______ complicated as it is serious. It is
complicated (2)______ much pollution is caused by things that benefit people. (3)______, exhaust from automobiles causes a
large percentage of air pollution. But the automobile (4)______ transportation for millions of people. Factories discharge much
(5)______ the material that pollutes the air and water, but factories give employment to a large number of people.
Thus, to end (6)_______ greatly reduce pollution immediately, people would have to ( 7)______ using many things
that benefit them. Most of the people do not want to do that, of course. But pollution can be (8)______ reduced in several
ways. Scientists and engineers can work to find ways to lessen the (9)______ of pollution that such things as automobiles and
factories cause. Governments can pass and enforce laws that (10)______ businesses and traffic to stop, or to cut down on
certain polluting activities.
1. A. as B. more C. less D. like
2. A. so B. while C. though D. because
3. A. Specific B. For example C. Such as D. Like
4. A. takes B. affords C. carries D. provides
5. A. about B. for C. of D. with
6. A. or B. and C. as well D. then
7. A. start B. continue C. stop D. go on
8. A. carefully B. unexpectedly C. gradually D. little
9. A. way B. figure C. number D. amount
10. A. forbid B. prevent C. request D. require
Read the following passage and choose the best answer. Write your answers in the numbered blanks provided below the
passage. (5pts)
Fort Collins certainly does its part to uphold Colorado’s reputation as one of the country’s most physically active states.
Recreational activities in Fort Collins  range from biking and hiking to snowboarding and kayaking. Thanks to its location at
the foothills of the Rocky Mountains and the banks of the Poudre River, and a robust lineup of city-run indoor exercise
facilities, Fort Collins residents have plenty of opportunities to get their hearts pumping. The city’s vibrant craft brewing scene
serves as a hard-earned reward for Fort Collins’ active locals.
Fort Collins has long prioritized the performance of its public school system, with efforts that have paid off in spades and
made the city one of the country's best places to live. Fort Collins' 41,680 students perform 3 to 19 percent higher than the
statewide average in all subjects; and on a national scale, Fort Collins pupils consistently are ahead of the game when it comes
to ACT and SAT college entrance exam averages.
Fort Collins has invested extensively in its local library system, cultural offerings, and public learning opportunities. The
city’s Fort Collins Museum of Discovery is a gorgeous hands-on learning space featuring an array of traditional and pop-
science exhibits that fascinate the young and old. The museum’s marquee attraction, the OtterBox Digital Dome Theatre,
screens a variety of nationally and internationally release science and nature films on a 35-foot dome screen.
Rounding out the city’s cultural, academic, and natural attractions is Fort Collins’ notable slate of industry because it’s
regarded as an engine of economic growth and stability for northern Colorado. Fort Collins enjoys the presence and job
provision of Fortune 500 luminaries. Businesses in Fort Collins find strong support from local residents and city leaders.
(Adapted from https://livability.com)
Question 1. Which could be the best title of the passage?
A. Fort Collins: Home Base for Outdoor Enthusiasts
B. Fort Collins: The Worth-living City
C. Proud of Education in Fort Collins
D. Fort Collins and Its Economic Policies
Question 2. The word "their" in paragraph 3 refers to ______.
A. foothills’ B. banks’ C. facilities’ D. residents’
Question 3. The word " release " in paragraph 3 is closest in meaning to ______.
A. broadcast B. absorb C. give freedom D. transmit
Question 4. According to the passage, why is Fort Collins a significant industry?
A. Residents and city officials are supportive of businesses
B. It’s famous for its agriculture.
C. It’s known as the driver of economic development in northern Colorado.
D. It lacks companies that provide employment.
Question 5. Which of the following is NOT TRUE about Fort Collins’ education as stated in the passage?
A. It has placed a premium on its state school system for a long time.
B. In all subjects, students surpass the state record average by 3 to 19 %
C. Students always outperform at entrance exam averages.
D. Students don’t have any place to study outside the school.
Part 3: Read the following passage and choose the correct answer to each question.
Write your answers on the answer sheet. (10 pts)
People have been donating blood since the early twentieth century to help accident victims and patients undergoing surgical
procedure. Usually a pint of whole blood is donated, and it is then divided into platelets, white blood cells and red blood cells.
People can donate blood (for red blood cells) about once every two months.
Transfusing the blood from the donor to the recipient is straightforward. It involves taking the blood from the donor’s arm vein
by means of hypodermic syringe. The blood flows through a plastic tube to a collection bag or bottle that contains sodium
citrate, which prevents the blood from clotting.
When the blood is given to the patient, a plastic tube and hypodermic needle are connected to the recipient’s arm. The blood
flows down from the container by gravity. This is a slow process and may last as long as two hours to complete the infusion of
blood to the recipient. The patient is protected from being infected during the transfusion. Only sterile containers, tubing and
needles are used, and this helps ensure that transfused or stored blood is not exposed to disease causing-bacteria.
Negative reactions to transfusion are not unusual. The recipient may suffer an allergic reaction or be sensitive to donor
leukocytes. Some may suffer from an undetected red-cell incompatibility. Unexplained reactions are also fairly common.
Although they are rare, other causes of such negative reactions include contaminated blood, air bubbles in the blood,
overloading the circulatory system through administration of excess blood, or sensitive to donor plasma or platelets.
Today, hospitals and blood banks go to great lengths to screen all blood donors and their blood. All donated blood is routinely
or rigorously tested for diseases, such as HIV (which causes AIDS), hepatitis B, and syphilis. When the recipient is a newborn
or an infant, the blood is irradiated to eliminate harmful elements. Donated blood is washed and the white blood cells and
platelets are removed.
Storing the blood sometimes requires a freezing process. To freeze the red blood cells, a glycerol solution is added. To
unfreeze the glycerol is removed. The ability to store blood for long periods has been a boon to human health.
1. Which of the following words is closest in meaning to the word “donating” in line 1?
A. Adorning B. Giving C. Taking D. Distributing
2. In line 3 the word “it” refers to
A. accident victims B. surgical procedures
C. a pint of whole blood D. surgery patients
3. According to the passage, how often can people donate blood for red blood cells
A. Every four months B. Every three months
C. Every two months D. Every month
4. Where in the passage is the best place for the following sentence?
Inserting the needles to the recipient’s arm causes little pain.
A. After the last sentence in the first paragraph B. After the word “syringe” in paragraph 2
C. After the word “arm” in paragraph 3D. After the word transfusion in paragraph 3
5. Which sentence in paragraph 2 explains how clotting is prevented in the blood container.
A. The first sentence B. The second sentence
C. The third sentence D. None of the above
6. All of the following are mentions as negative reactions to transfusion EXCEPT
A. allergies B. red-cell incompatibility
C. air bubbles in the blood D. sensitive to donor leukocytes
7. What answered choice is closest in meaning to the word “undetected” in line 17?
A. Not wanted B. Not captured C. Not found D. Not illustrated
8. Look at the phrase “Go to great lengths to screen” in paragraph 5. Choose the word that has the same meaning.
A. Routinely B. Rigorously C. Irradiated D. Removed
9. Based on the information in the passage, what can be inferred about blood transfusion to infants and newborns ?
A. It is as rigorously tested as blood for adults.B. It is treated with radiant energy
C. It is not treated differently from adultsD. It is not dangerous for children
10. What does the author implies in the passage?
A. Transfusing blood is a dangerous process B. Storing blood benefits mankind
C. Clotting cannot be preventedD. Freezing blood destroys platelets.

Đề 24
Put each word in brackets into an appropriate form.
1. The woman is getting older. She is now in the __________ year of her age. (LIGHT)
2. Chemicals are easily __________ if they are laid open in the atmosphere for a long time. (ACTION)
3. My ________ is the unique thing that distinguishes me from others. (MARK)
4. The advent of computers was one of the _________ milestones in the mankind's history. (NOTE)
5. His condition was so serious _________ as he had to stay in hospital for a month. (MUCH)
6. Children who grow up in peace tend to be more ________ than those who grow up in (ADAPT)
war.
7. Many people nowadays want to indulge themselves in ________ industries such as (KNOW)
finance and the like.
8. I met my old friend at the supermarket this morning, which was quite ________. (FORTUNE)
9. A(n) ________ is the one who turns into wolf during full moon. (WOLF)
10. Nitrogen oxide is a(n) ________ compound of oxygen and nitrogen. (ATOM)
Complete the sentences below, using a phrasal verb from the box in its correct form.
jealous of by heart for a change content with in common
afraid of in captivity similar to in charge good at
1. Gold is .......................... brass in color.
2. I must speak to someone responsible. Who’s .......................... here?
3. She is very ............................... swimming.
4. Jane’s new friend is a Scorpio, and he likes cats, so they have a lot ...........................
5. He was ................................ his brother’s success.
6. Zoos were very popular, but now many people think it’s wrong to keep animals
...................................
7. As there’s no cream, so we have to be .............................. milk.
8. I’m tired of staying at home on Sundays. Let’s go out ...................................
9. I’m very ................................ snakes. I can’t go near them.
10. At school we used to have to learn bits of Shakespeare ..................................
There are 10 errors in the following passage. Identify and correct them. The first has been done for you.
Crime preventing is as crucial in the workplace as it is in the home or neighborhood. Reducing crime is as much a part of good
management as prompt delivery, good staff relations, and other acceptable management functions. Losses from shops through
shoplifting are extremely high and ultimately, those losses are payment for by all of us in high prices. There are many
opportunities for shopkeepers themselves to reduce shoplifting. As with all types of criminal, prevention is better than cure.
The best
deterrent is the present of staff properly trained in how to identify potential shoplifters. There are also many secure devices
now available. Video camera surveillance is a popular system, even with quite small retailers. In clothes shopping, magnetic
tag marking systems that set off an alarm if they are taken out of the shop have proved their worth. However, there are many
simpler measure that retailers should consider. Better lighting and ceiling-hung mirrors can help staff to watch all parts of the
display area. Similarly, simply arrangement shelves and display units to allowing clear fields of vision is a good deterrent.
Line 1
Line 2
Line 3
Line 4
Line 5
Line 6
Line 7
Line 8
Line 9
Line 10
Line 11
Line 12
Line 13
Line 14
Line 15
Line 16
- Read the passage below and choose the correct word or phrase to complete each of the blanks. (5 pts)
Allergies
Put simply, an allergy is a disorder in which the body over- reacts to harmless substances which in normal circumstances
should not produce any reaction at all. An allergy can occur in almost (1) ______ part of your body, and can (2) ______
caused by just about anything by just about anything. But mainly, (3) ______ become evident on parts of the body directly
exposed (4) ______ the outside world. Certain allergies occur only at certain times of the year, while (5) ______ are there
all the time. Those (6) ______ occur all the year round and are probably caused by something you come into contact (7)
______ everyday of your life, some seemingly harmless object such as your deodorant or the pillow you lie on each night.
Allergies can occur at any time during your life, (8) ______ usually do so before your fortieth birthday. Sometimes the
symptoms are (9) ______ slight you do not even know you have an allergy, and it may take years for an allergy to become
noticeable. Sometimes it comes and (10) ______ for no apparent reason and with no regularity-
Finish each of the following sentences in such a way that it means the same as the original one(s).
1. I am having a lot of trouble now because I lost my passport last week.
→ If I ..........................................................................................................................................................................
2. The hurricane blew the roof off the house.
→ The house .............................................................................................................................................................
3. He was sentenced to six months in prison for his part in the robbery.
→ He received a .........................................................................................................................................................
4. That rumor about the politician and the construction contract is absolutely false.
→ There is
5. I thought about what had happened all those years before.
→ I cast .....................................................................................................................................................................
6. There aren’t many other books which explain this problem so well.
→ In few other books..................................................................................................................................................
7. Don’t you think we should ask the price?
→ Hadn’t.....................................................................................................................................................................?
8. Doris tiptoed up the stairs because she didn’t want to wake anyone up.
→ To ........................................................................................................................................................................... .
9. I’m sure it wasn’t Mr. Bill you saw because he’s in New York.
→ It can’t ...................................................................................................................................................................
10. The collision didn’t damage my car much.
→ Not a great ............................................................................................................................................................
Rewrite the sentences
1. You must accept the fact that she has left you. RESIGN
.....................................................................................................................................................................................
2. His latest theory is in complete opposition to mainstream thinking. FACE
.....................................................................................................................................................................................
3. Sam was really anxious, waiting to see if he had got a place in the cricket team.
TENTERHOOKS
.....................................................................................................................................................................................
4. The loss of the account was not our fault. BLAME
.....................................................................................................................................................................................
5. You shouldn’t judge people by their appearance. BOOK
.....................................................................................................................................................................................
" Some people feel that certain workers like nurses, doctors, and teachers are undervalued and should be paid more"
How far do you agree? Give reasons for your answer and include any relevant examples from your own knowledge or
experience.
You should write at least 150 words
Mark the letter A, B, C, or D on your answer sheet to indicate the word whose underlined part differs from the other
three in pronunciation in the following question.
1. A. breath B. break C. thread D. tread
2. A. believes B. pencils C. contents D. tables
3. A. ragged B. wicked C. naked D. packed
4. A. cherish B. chorus C. chaos D. scholar
5. A. hysteria B. hypocrite C. hypocrisy D. hydroplane
Mark the letter A, B, C, or D on your answer sheet to indicate the word that differs from the rest in the position of the main
stress in each of the following questions.
1. A. offer B. persuade C. apply D. reduce
2. A. adventure B. advantage C. advertise D. adverbial
3. A. irrational B. characterise C. facility D. variety
4. A. therapeutic B. conscientious C. fanaticism D. orthographic
5. A. interpolate B. acclimatize C. cannibalize D. demystify
CAUHOI
1. My mind went ________ when the official asked me my phone number- I couldn't remember it at all.
A. empty B. clear C. blank D. vacant
2. The life ________ of individuals in developed countries increases dramatically every decade.
A. expectation B. exception C. exertion D. expectancy
3. She believes that all countries should ________ the death penalty as it is inhumane.
A. put down to B. catch up on C. get down to D. do away with
4. I'm amazed that this game ever ________ - it is so silly!
A. took in B. caught on C. took up D. caught by
5. Their research into the causes of cancer promises to break new ________ in the field and possibly lead to a cure.
A. earth B. ground C. soil D. land
6. Different measures have been ________ to help the homeless.
A. make B. taken C. done D. conducted
7. The noisy music ________ the students from their test.
A. distracted B. disrupted C. dissolved D. disqualified
8. Health food is now ________ in popularity.
A. growing B. raising C. getting D. flying
9. It was ________ of Harry to arrive late and then try to dominate the meeting.
A. common B. usual C. ordinary D. typical
10. The match will be screened on ITV with ________ commentary by Andy Gray.
A. lively B. live C. alive D. living
Complete the following passage by choosing the correct option (A, B, C or D) to fill in blanks. (5pts)
Viewed from the outside (1) ________, the Houses of Parliament look impressive. The architecture gives the place a
traditional look, and the buildings are sandwiched between a busy square and the river, making them a (2) ________between
the country house of an eccentric duke and a Victorian railway station. You have only to learn that the members (3) ______ to
each other as ‘The Honorable Member to (4) ______ the picture of a dignified gentlemen’s club, with of course a few ladies to
(5) _______ the numbers. Sadly, over the past few years first radio, and now television, have shown the general public, who
are (6) ______ the electorate, what in fact goes on when bills are discussed and questions are asked. The first obvious fact is
that the chamber is very rarely full, and there may be only a handful of members present, some of whom are quite clearly
asleep, telling jokes to their neighbor, or shouting like badly-behaved schoolchildren. There is not enough room for them all in
the chamber in any (7) _______, which is a second worrying point. Of course, television does not follow the work of
committees, which are the small discussions groups that do most of the real work of the House. But the (8) ______ impression
that voters receive of the workings of government is not a good one. To put it (9) _______, parliament looks disorganized, is
clearly behind the time and seems to be filled with bores and comedians. This is presumably why members (10) _______ for
so long the efforts of the BBC to broadcast parliamentary matters on television.
1. A. likewise B. at least C. nevertheless D. as well
2. A. mixture B. combination C. cross D. match
3. A. call B. refer C. speak D. submit
4. A. finalize B. end C. conclude D. complete
5. A. take away B. bring about C. make up D. set in
6. A. after all B. anyway C. even D. furthermore
7. A. point B. way C. matter D. case
8. A. total B. broad C. overall D. comprehensive
9. A. bluntly B. shortly C. directly D. basically
10. A. prevented B. checked C. defied D. resisted
Read the following passage and choose the best answer for each question. Identify your answer by writing the
corresponding letter A, B, C or D on your answer sheet. (5pts)
When we accept the evidence of our unaided eyes and describe the Sun as a yellow star, we have summed up the most
important single fact about it-at this moment in time.
It appears probably, however, that sunlight will be the color we know for only a negligibly small part of the Sun's history.
Stars, like individuals, age and change. As we look out into space, we see around us stars at all stages of evolution. There are
faint blood-red dwarfs so cool that their surface temperature is a mere 4,000 degrees Fahrenheit, there are searing ghosts
blazing at 100,000 degrees Fahrenheit and almost too hot to be seen, for the great part of their radiation is in the invisible
ultraviolet range. Obviously, the "daylight" produced by any star depends on its temperature; today(and for ages to come) our
Sun is at about 10,000 degrees Fahrenheit, and this means that most of the Sun's light is concentrated in the yellow band of the
spectrum, falling slowly in intensity toward both the longer and shorter light waves.
That yellow "hump" will shift as the Sun evolves, and the light of day will change accordingly. It is natural to assume that
as the Sun grows older, and uses up its hydrogen fuel-which it is now doing at the spanking rate of half a billion tons a second-
it will become steadily colder and redder.
(Adapted from A collection of TOEFL Reading comprehension 4)
Question 1. What is the passage mainly about?
A. Faint dwarf stars B. The evolutionary cycle of the Sun
C. The Sun's fuel problem D. The dangers of invisible radiation
Question 2. The word “searing” in the 2nd paragraph is CLOSEST in meaning to_____
A. so strong B. far C. negligible D. unaided
Question 3. What does the word "it" in the last paragraph refer to_____ ?
A. yellow "hump" B. day C. Sun D. hydrogen fuel
Question 4. Why are very hot stars referred to as "ghosts"?
A. They are short- lived. B. They are mysterious.
C. They are frightening. D. They are nearly invisible
Question 5. Which of the following sentences is NOT true according to the passage?
A. The Sun’s history is still mysterious. B. All the stars depend on the Sun for light.
C. The Sun is too hot to be seen by naked eyes. D. The sun never uses up its hydrogen fuel.
Part 3: Read the following passage and choose the correct answer to each question.
Write your answers on the answer sheet. (10 pts)
A number of factors related to the voice reveal the personality of the speaker. The first is the broad area of communication,
which includes imparting information by use of language, communicating with a group or an individual, and specialized
communication through performance. A person conveys thoughts and ideas through choice of words, by a tone of voice that is
pleasant or unpleasant, gentle or harsh, by the rhythm that is inherent within the language itself, and by speech rhythms that are
flowing and regular or uneven and hesitant, and finally, by the pitch and melody of the utterance. When speaking before a
group, a person's tone may indicate unsureness or fright, confidence or calm. At interpersonal levels, the tone may reflect
ideas and feelings over and above the words chosen, or may belie them. Here the speaker's tone can consciously or
unconsciously reflect intuitive sympathy or antipathy, lack of concern or interest, fatigue, anxiety, enthusiasm or excitement,
all of which are usually discernible by the acute listener. Public performance is a manner of communication that is highly
specialized with its own techniques for obtaining effects by voice and /or gesture. The motivation derived from the text, and in
the case of singing, the music, in combination with the performer's skills, personality, and ability to create empathy will
determine the success of artistic, political, or pedagogic communication.
Second, the voice gives psychological clues to a person's self-image, perception of others, and emotional health. Self-image
can be indicated by a tone of voice that is confident, pretentious, shy, aggressive, outgoing, or exuberant, to name only a few
personality traits. Also the sound may give a clue to the facade or mask of that person, for example, a shy person hiding behind
an overconfident front. How a speaker perceives the listener's receptiveness, interest, or sympathy in any given conversation
can drastically alter the tone of presentation, by encouraging or discouraging the speaker. Emotional health is evidenced in
the voice by free and melodic sounds of the happy, by constricted and harsh sound of the angry, and by dull and lethargic
qualities of the depressed.
1. What does the passage mainly discuss?
A. The function of the voice in performance B. The production of speech
C. The connection between voice and personality D. Communication styles
2. What does the author mean by staring that, "At interpersonal levels, tone may reflect ideas and feelings over and above
the words chosen" 
A. The tone of voice can carry information beyond the meaning of words.
B. A high tone of voice reflects an emotional communication.
C.  Feelings are more difficult to express than ideas.
D.  Feelings are expressed with different words than ideas are.
3. The word "Here" in line 10 refers to _______.
A. ideas and feelings B.  the tone
C.   words chosen D. interpersonal interactions
4. The word "derived" in line 14 is closest in meaning to _______.
A. discussed B. registered C. prepared D.  obtained
5. Why does the author mention "artistic, political, or pedagogic communication" in line 13?
A. As examples of basic styles of communication
B. As examples of public performance
C.  To introduce the idea of self-image
D.  To contrast them to singing
6. According to the passage, an exuberant tone of voice, may be an indication of a person's _______.
A. personality B. general physical health
C. ability to communicate D. vocal quality
7. According to the passage, an overconfident front may hide _______.
A. strength B. friendliness C.  shyness D. hostility
8. The word "drastically" in line 18 is closest in meaning to _______.
A. severely B. easily C. exactly D. frequently
9. The word "evidenced" in line 19 is closest in meaning to _______.
A. indicated B. repeated C. exaggerated D. questioned
10. According to the passage, what does a constricted and harsh voice indicate?
A. Depression B. Anger C. Lethargy D.  Boredom

You might also like